1000 CR Test 8 Q1

This topic has expert replies
Senior | Next Rank: 100 Posts
Posts: 83
Joined: Mon Oct 22, 2007 4:44 am
Thanked: 1 times

1000 CR Test 8 Q1

by its_me07 » Sat Feb 23, 2008 5:21 am
A drug that is highly effective in treating many types of infection can, at present, be obtained only from the bark of the ibora, a tree that is quite rare in the wild. It takes the bark of 5,000 trees to make one kilogram of the drug. It follows, therefore, that continued production of the drug must inevitably lead to the ibora’s extinction.
Which of the following, if true, most seriously weakens the argument above?
(A) The drug made from ibora bark is dispensed to doctors from a central authority.
(B) The drug made from ibora bark is expensive to produce.
(C) The leaves of the ibora are used in a number of medical products.
(D) The ibora can be propagated from cuttings and grown under cultivation.
(E) The ibora generally grows in largely inaccessible places.


i chose d but the OA given in 1000 doc i s
E

Master | Next Rank: 500 Posts
Posts: 124
Joined: Thu Aug 23, 2007 5:11 am
Thanked: 2 times

by gmatguy16 » Sat Feb 23, 2008 10:42 am
i agree with d .

User avatar
Legendary Member
Posts: 566
Joined: Fri Jan 04, 2008 11:01 am
Location: Philadelphia
Thanked: 31 times
GMAT Score:640

.

by AleksandrM » Tue Feb 26, 2008 10:20 am
D goes outside of the scope intended. However, as E indicates, if the plant is largely inaccessible then it will significantly reduce the chance of it going extinct.

Whenever you are facing a CR question and you are down to two likely answers, ask yourself which is the most relevant to the ideas, etc mentioned in the passage. For example, the passage does not say anything about growing the ibora, nor does it hint at the likelihood of there being alternative ways, other than gathering the plant in the wild, to get your hands on the ibora. The passage tells you that it can currently be gathered in its natural form and it follows that if E is true, then the arguement is weakened.

Junior | Next Rank: 30 Posts
Posts: 28
Joined: Fri Dec 14, 2007 7:58 am
Thanked: 1 times

by Riggz » Tue Feb 26, 2008 10:55 am
The OA given is incorrect. I did this question last night from OG 10.

The correct answer is D.

If the Ibora plant can be grown under cultivation it is unlikely the Ibora plant will become extinct, thus weakening the conculsion.

Just because the Ibora plant is inacessible does not mean it will reduce the chances of it going extinct especially when it will be used for continued production of a drug.

User avatar
GMAT Instructor
Posts: 3225
Joined: Tue Jan 08, 2008 2:40 pm
Location: Toronto
Thanked: 1710 times
Followed by:614 members
GMAT Score:800

by Stuart@KaplanGMAT » Tue Feb 26, 2008 12:21 pm
To make the leap from "it takes a lot of trees to make the drug" to "the tree will become extinct", the author has to be assuming that the supply of trees is finite.

To weaken the argument, we look for an alternative to this assumption. Any answer choice that suggests that the supply of trees isn't as limited as the author says will weaken the conclusion.

(D) says that we can grow the tree at tree farms (or greenhouses, or whatever). If we can grow the tree ourselves without touching the trees in the wild, then there's no reason why the tree should become extinct.

Choose (D).

(answer pasted from https://www.beatthegmat.com/ibora-t8151.html)
Image

Stuart Kovinsky | Kaplan GMAT Faculty | Toronto

Kaplan Exclusive: The Official Test Day Experience | Ready to Take a Free Practice Test? | Kaplan/Beat the GMAT Member Discount
BTG100 for $100 off a full course

User avatar
Legendary Member
Posts: 566
Joined: Fri Jan 04, 2008 11:01 am
Location: Philadelphia
Thanked: 31 times
GMAT Score:640

by AleksandrM » Tue Feb 26, 2008 6:23 pm
I stand corrected. The funny thing is that I answered that question correctly the first time I saw in the OG. I knew it looked familiar. My apologies.

Newbie | Next Rank: 10 Posts
Posts: 4
Joined: Mon Apr 07, 2008 6:28 am

by nikhil_havele » Mon Apr 07, 2008 10:26 am
If you ask me, answering strength/weakening questions is the easiest thing to do. These are the steps one need to follow:
Lets take the above question:

STEP 1 : Write down the claim in your own simple words.Here the claim is " continous production of drug will lead to extinction of ibora"

STEP2 : Write down the sentence in your own words which contradicts the claim. In this case we can write down following sentence " continous production of drug will NOT lead to extinction of ibora"
Was that easy enough?????

STEP3: Look for the options. In this case you will see Option D states that ibora will not become extinct because it can be cultivated.


**All the above mentioned 3 steps should be done very fast within 1 to 1 1/2 mins.

Senior | Next Rank: 100 Posts
Posts: 58
Joined: Wed Apr 16, 2008 5:51 pm

by gmat765 » Fri Apr 18, 2008 5:50 pm
I will choose D

Newbie | Next Rank: 10 Posts
Posts: 2
Joined: Sun Jan 06, 2008 7:12 am

to > itsme_07

by zlickfish » Sat Apr 19, 2008 2:11 am
A drug that is highly effective in treating many types of infection can, at present, be obtained only from the bark of the ibora, a tree that is quite rare in the wild. It takes the bark of 5,000 trees to make one kilogram of the drug. It follows, therefore, that continued production of the drug must inevitably lead to the ibora’s extinction.

Which of the following, if true, most seriously weakens the argument above?


(A) The drug made from ibora bark is dispensed to doctors from a central authority.
[wrong, out of scope]

(B) The drug made from ibora bark is expensive to produce.
[wrong, out of scope]

(C) The leaves of the ibora are used in a number of medical products.[wrong, out of scope, irrelevant]

(D) The ibora can be propagated from cuttings and grown under cultivation.[right answer, because the population of this tree can be increased and one can thereby save it from extinction; weakens the conclusion in the question ]

(E) The ibora generally grows in largely inaccessible places. [wrong, absolutely out of scope]
(':P')

The answer is D. any other answer would be a typo.

Master | Next Rank: 500 Posts
Posts: 139
Joined: Sun Nov 09, 2008 12:04 am
Thanked: 3 times
GMAT Score:620

by Rashmi1804 » Sun Apr 05, 2009 4:26 am
IMO D

I think people who drilled down the following reasoning picked E :

" If Ibora is largely inaccessible " then drug-makers cannot use it, leading to it's extinction"...right ??

BUT!! option E is so generalized...." ibora generally grows in largely inaccessible places"---------If that is the case...THEN how did the drug-makers even make the sample of drugs.
Hence false!! : - )

Master | Next Rank: 500 Posts
Posts: 100
Joined: Tue Aug 05, 2014 3:58 am

by vanessa.m » Sun May 15, 2016 2:13 am
Anothewr vote for D

User avatar
Newbie | Next Rank: 10 Posts
Posts: 4
Joined: Thu Aug 07, 2014 3:34 pm

by alok322 » Sun Jun 12, 2016 2:40 am
Hi, Is " A Kilogram of medicine is all that is required to cure the disease for the whole world and there are more than 1 lakh plants in the forest" a valid weakener?